Sie sind auf Seite 1von 14

CSE(P)-2008 1 of 14

PHYSICS
1. Consider the following statements: MR 2
A body, initially at rest, is acted upon by an a. 0
4
external force which is constant in magnitude.
MR 2
The gain in kinetic energy of the body at a b. 0
given instant 2
1. is directly proportional to the distance MR 2
covered by it. c.
2
2. is directly proportional to the time elapsed. d. None of the above
3. is independent of the time elapsed. Ans. c
Which of the statements given above is/are 5.
correct?
a. 1 only
b. 2 only
c. 1 and 2
d. 1 and 3
Ans. a
2. A particle of mass m falls from a height h, and
A book of mass m is held against a wall by
bounces in elastically on the ground. In this
pressing a finger against it, as shown in the
process, the energy reduces by a factor F < 1.
figure above. The coefficient of friction
If vi and vf are the velocities before and after
the collision respectively, which one of the between the wall and the book is . If the
following is correct ? finger pressure is slowly released, what is the
force applied by the finger on the book when
a. the book just starts slipping?
a. mg
b. b. g
c. c. mg/
d. m/g
d. Ans. c
Ans. b 6. A star of mass M has ten planets, all of the
3. A particle motion in the x-y plane is governed same mass but at different distances from the
by its position vector r = a sin t i + b cot star. How are potential energy (Ep) kinetic
t j, where a, b and co are positive constants energy (Ek), and angular momentum (L)
and a > b. What is the trajectory of the particle affected as the distance of a planet from star
? increases ?
a. A parabola a. Ek decreases; Ep and L increase
b. A hyperbola b. Ep decrease; Ek and L increase
c. An ellipse c. Ek decreases: Ep increases, L remains
d. A circle constant
Ans. c d. Ek increases; Ep decreases, L remains
4. constant
Ans. c
7. Imagine a particle at rest on the top of a
frictionless sphere of radius 5 m. When the
particle is displaced slightly, it starts sliding,
without rolling, down the sphere. It makes an
angle with the horizontal axis just before
leaving the sphere. What is the speed of the
What is the moment of inertia of a section of particle at the position, where it leaves the
mass M of a right circular cylinder of radius R sphere ? (g = 10 m/s2)
that subtends an angle 0 at the origin, as
a. 10 1 + cos m / s
shown in the figure, when the reference axis is
at the origin and perpendicular to the section? b. 10 (1 + sin ) m / s
c. 10 (1 cos ) m / s
2 of 14
d. 10 (1 sin ) m / s 13. Consider the following statements for
a particle moving in an elliptic orbit under the
Ans. c
influence of a central force:
8. Range of a projectile is 48 m. What is the
1. The radius vector covers equal area in
horizontal distance covered at a position where
equal time.
the kinetic energy is minimum?
2. The motion takes place in a plane.
a. 48 m
3. The angular momentum is a constant of
b. 36m
motion.
c. 24 m
Which of the statements given above are
d. 12m
correct?
Ans. c
a. 1 and 2 only
9. A stone of mass m, tied to the end of string is
b. 2 and 3 only
whirled around in a horizontal circle. The
c. 1 and 3 only
length of string is reduced gradually keeping
d. 1, 2 and 3
the angular momentum of the stone about the
Ans. d
centre of circle constant. The tension in the
14. Consider the following statements for a
string is proportional to rn where r is the
conservative force:
instantaneous radius of circle. What is the
1. Work done by the conservative force on a
value of n? (Neglecting force of gravity)
particle moving between two points is
a. 2
independent of the path.
b. 3
2. Force of gravity is a conservative force.
c. -2
Which of the statements given above is/are
d. -3
correct?
Ans. d
a. 1 only
10. Consider a simple harmonic motion in 2-
b. 2 only
dimensions under the influence of a
c. Both 1 and 2
conservative force. The potential energy of the
d. Neither 1 nor 2
oscillator is given by V =
(
k x 2 + y2 ) , where Ans. c
2 15. Consider the following statements:
k is a positive constant. What is the magnitude 1. Volume of fluid flow in a tube in which a
of the applied force? constant pressure difference is maintained
a. (k/2) (x2 + y2) increases when the temperature of the
b. k (x2 + y2) fluid increases.
c. (k/2) (x2 + y2)1/2 2. Density of the fluid decreases with the
d. k(x2 + y2)1/2 increase in temperature and there is more
Ans. d fluid volume per unit mass at high
11. A thin circular ring of radius r and mass m temperature.
rolls on a horizontal surface without slipping. Which of the statements given above is/are
The speed of its centre is v. What is the kinetic correct?
energy of the ring? a. 1 only
a. mv2/2 b. 2 only
b. mv2 c. Both land 2
c. 2 mv2 d. Neither 1 nor 2
d. mv2/3 Ans. c
Ans. b 16. A reservoir containing water has an orifice
12. A man will feel weightlessness on the surface located 5 m below the free surface level.
of equator if earth has kinetic energy (M and R Assuming no wasted energy, what is the speed
are the mass and radius of earth, respectively) of discharge of water through the orifice ? (g =
equal to which one of the following? 10 m/s2)
5MgR a. 25 m/s
a. b. 5 m/s
2 c. 75 m/s
3MgR d. 10 m/s
b.
2 Ans. d
MgR 17. Rain drops, assumed to be spheres of radii R,
c.
2 2R, 3R, 4R .. , fall through the air. Which
d. Mg R one of the following statements is correct ?
Ans. d The terminal velocities of
a. all of them will be same
3 of 14
b. drops of radius R will be maximum A particle subjected to two SHMs of
c. drops of radius R will be minimum the same frequency but in two orthogonal
d. drops of radius 2Rwill be eight times the directions, vibrates along the line AOB as
terminal velocities of drops of radius R shown in the figure. What is the phase
Ans. c difference between these two vibrations?
18. Light starting from a diametrically endpoint of a. 0
a semi-spherical surface reaches to the b. /2
opposite diametrically end-point after being c.
reflected from the inner surface of the semi- d. 3/2
sphere. If the actual path followed by the light Ans. c
is maximum, what would be the angle which 24. Consider the following:
light ray should make with the diameter at the If the Lissajous figure obtained by a
starting end? combination of two simple harmonic motions
a. /6 is given by an ellipse in general, then the
b. /4 motions are
c. /3 1. parallel to each other
d. None of the above 2. at right angle to each other
Ans. b 3. with equal frequencies
19. Two particles are moving in opposite Which of the above is/are correct?
directions, each with a speed 0.9 c in a. 1 and 3
laboratory frame of reference. What is the b. 2 and 3
velocity of one particle with respect to other ? c. 2 only
a. Zero d. 1 only
b. 0.9 c Ans. b
c. 0.18 c 25. The equation of a damped harmonic oscillator
d. 0.994 c is given by z + 2 z + 2z =0. Consider the
Ans. d following
20. How will a square object moving with 1. 2 > 2
relativistic speed 0.6 c appear to an observer? 2. 2 = 2
a. Square 3. 2 < 2
b. Rectangle What is the correct sequence corresponding to
c. Triangle critically damped, under-damped and and
d. Circular over-damped motions respectively?
Ans. b a. 2, 1, 3
21. If the relativistic mass of a particle is twice its b. 2, 3, 1
rest mass, then what is the ratio of its speed to c. 3, 1, 2
that of light? d. 3, 2, 1
a. 3/2 Ans. b
b. 1 / 2 26. Power transmitted by a wave of frequency 600
c. 1/2 Hz on a string with tension T is P. If the
d. 1/4 frequency is reduced to 300 Hz, what will be
Ans. a the power transmitted ?
22. The kinetic energy of a body is twice its rest a. P/4
mass energy. What is the ratio of relativistic b. P/2
mass to rest mass of the body? c. 2P
a. 3 d. 4 P
b. 1 Ans. a
c. 1/2 27. Two bottles are of equal volume and height
d. 2 but have necks of different lengths and same
Ans. a area of cross-section. Sound is produced by
23. blowing across the mouth of each bottle.
Which one of the following statements is
correct in respect of the frequency of sound
produced ?
a. Long necked bottle produces higher
frequency sound
b. Short necked bottle produces higher
frequency sound
4 of 14
c. Both bottles produce the sound of same 32. What is the power of a combination of
frequency two lenses of focal lengths f1 and f2 at a
d. None of the above separation (f1 + f2) ?
Ans. c a. (f1 + f2)
28. The refractive index of a medium varies, when b. (f1 - f2)
0 x 1 as (x) = 1.5(2-x) where 1.5 is a c. 0
number having dimension L-1. What is the d.
optical path of a ray moving from x = 0 to x = Ans. c
1? 33. Let S0 and Si be the object and image distances
a. 1.5 in a system of two media with refractive
b. 2.25 indices 0 and i respectively. The boundary
c. 3.0 surface between the two media has a spherical
d. 0.75 curvature. Consider the paraxial optics case.
Ans. b What will be magnification in the image of an
29. object due to the spherical surface ?

a.

b.

c.

Consider a light ray traversing 3 layers of


d.
different refractive indices separated by air
gap. The refractive indices are 1, 2, 3, The
incident ray and emergent ray are parallel. Ans. b
Which one of the following conditions should
be applicable? 34. Which one of the following imaging be totally
a. 1, 2, 3, can have any values free of chromatic systems will aberration ?
b. 1> 2> 3 a. A very thin convex lens
b. A very thin concave lens
c. 1< 2 < 3
c. A back coated concave mirror
d. 1= 3, 2 can have any value
d. A front coated concave mirror
Ans. d
Ans. d
30. In a prism of prism angle 35, a light ray is
35. Consider the following statements The
incident on one surface with angle of
spherical aberration in a refracting type
incidence 30 and it emerges from the other
system, e.g., a microscope, can be reduced by
surface making an angle 65 with the normal
1. using a thick lens.
to that surface. What is the angle of deviation?
2. increasing the aperture.
a. 0
3. dividing the total deviation of rays equally
b. 65
over a large number of refracting surfaces.
c. 60
4. making use of aplanatic points.
d. 70
Which of the statements given above is/are
Ans. *
correct?
31. A light ray passes through a glass plate of
a. 3 only
thickness (d) and refractive index (). How b. 3and4
much time will be taken by the light ray to c. 2and3
cross this glass plate? (Velocity of light in d. 1 only
vacuum is c) Ans. b
a. mc 36. A notebook has 200 pages and is 1 cm thick.
b. d/(c) D
c. d/c With respect to light of wavelength 5,000 A
d. c/d how many wavelengths will be contained in
Ans. c the thickness of one page ?
a. 10,000
b. 1,000
5 of 14
c. 100 43. In a two slit diffraction pattern, several
d. 10 bright and dark fringes are seen. For what path
Ans. c difference are the dark fringes formed ?
37. What is the wavelength of the plane a. Exactly zero
electromagnetic wave in SI units given by the b. Tending to infinity
expressions Ex = 0, Ey = 2 cos c. Integral multiple of wavelength
d. Odd multiple of half wavelength
Ans. d
a. 15 x 10-6 m 44. On what factors does the angular dispersion
b. 30 x 10-6 m d
of a plane transmission grating depend?
c. 30 x 10-5 m d
d. 30 x 10-4 m 1. Order of spectrum
Ans. b 2. Grating element
38. In Michelsons interferometer, if N fringes 3. cos
pass from the field of view on moving the Select the correct answer using the code given
movable mirror by a distance x, what is the below code
wavelength of light used ? a. 1 and 2 only
a. Nx b. 2 and 3 only
b. 2x/N c. 1 and 3 only
c. x/(2N) d. 1, 2 and 3
d. x/N Ans. d
Ans. b 45. For a given angle of incidence and diffraction
39. In a Youngs double slit experiment using two order, the number of lines of a plane grating is
wavelengths 1= 600 nm and 2= 400 nm, the doubled keeping the width of the grating the
mth bright fringe of 1 coincides with the nth same. How will the resolving power of the
bright fringe of 2. What are the possible value grating change?
of m and a. It will double
a. m = 2, 4, 8, ..; n = 3, 5, 7 .. b. It will increase three times
b. m = 2, 4, 6, ..; n = 3, 6, 9 .. c. It will decrease
c. m = 3, 9, 15, .; n = 2, 4, 6, .. d. It will remain the same
d. m = 3, 6, 9, .; n = 2, 4, 6, .. Ans. d
Ans. b 46. The material of a quarter wave plate has
40. In Newtons ring experiment, circular rings are refractive indices 0 and e for ordinary and
formed. Which one of the following extraordinary rays respectively. The
phenomena is this pattern due to? geometrical thickness of the plate will be
a. Interference by division of amplitude proportional to which one of the following ?
b. Interference by division of wavefront
c. Wavefront reconstruction a.
d. Multiple beam interference b.
Ans. a
41. Which one of the following optical elements c.
can be used in conjunction with a point source d.
to obtain a suitable wavefront for Fraunhofer Ans. c
diffraction in the laboratory ? 47. If a plate behaves as /2 plate for the light of
a. Concave lens D
b. Convex lens wavelength 4000 A , how will the same plate
c. Convex mirror D

d. Plane mirror behave for light of wavelength 8000 A A,


Ans. b assuming that the refractive index is the same?
42. A diffraction pattern is obtained using a beam a. /2 plate
of red light. What happens if the red light is b. /4 plate
replaced by blue light ? c. Plane plate
a. No change d. Nothing can be said unless the thickness
b. Diffraction bands become narrower and of plate is known
crowded Ans. b
c. Bands become broader and farther apart 48. A plane polarized beam of wavelength is
d. Bands disappear allowed to pass through an optically active
Ans. b crystal plate of thickness t, and a rotation of
is observed for the plane of vibration on
6 of 14
emergence. If and a and c are the refractive d. Tv (-1) = constant
indices of the crystal in the direction of the Ans. d
optic axis for anti- clockwise and clockwise 53. Consider the following statements relating to a
circular polarized light, which one of the heat reservoir:
following relations for the difference of 1. Total heat content in a reservoir is
refractive indices is correct ? considered as unchanged in a
thermodynamical process.
a. 2. Temperature of reservoir remain
unchanged even after absorbing or
b. supplying large amount of heat.
3. A black body is an example of heats.
reservoir.
c. Which of the statements given above are
correct?
d. a. 1, 2 and 3
Ans. b b. 1 and 2 only
49. What is the order of the diameter of a typical c. 2 and 3 only
single fibre used in a communication fibre d. 1 and 3 only
bundle ? Ans. b
a. 100 m 54.
b. 10 m
c. 1 m
d. 0.1 m
Ans. d
50. A milli Watt He-Ne laser beam is brighter than
a 500 W bulb. Which of the following is
mainly responsible for the high brightness of a
laser beam ?
a. Monochromaticity 2 moles of an ideal monoatomic gas perform a
b. Polarization cycle shown in figure given above.
c. High spatial and temporal coherence Temperatures at points A, B, C and D are 300
d. Power K, 400 K, 700 K and 550 K, respectively.
Ans. c What is the work done by the gas ? (R = 83 J
51. mol-1 K-1)
a. 622.5 Joule
b. 1245 Joule
c. 2490 Joule
d. None of the above
Ans. d
55.

The existence/coexistence of solid, liquid and


vapor phases of a pure substance under
different thermodynamical conditions is
shown above.
Which variables are plotted along the y-axis
and x-axis, respectively? AB isochoric heating
a. Pressure and volume BC : adiabatic expansion
b. Pressure and temperature CA isothermal compression
c. Volume and pressure One mole of an ideal gas is taken round the
d. Volume and temperature cycle ABCA as in the diagram shown above.
Ans. c What is the efficiency of a heat engine
52. In which of the following processes for an operating on this cycle ?
ideal gas, the specific heat of the gas is zero ?
a. T = constant
a.
b. v = constant
c. T/v = constant
7 of 14
b. a. A gas can be liquefied by
application of pressure alone when the
temperature is above Tc
b. Densities of liquid and vapour states are
equal at Tc
c.
c. The horizontal portion in P-v curve
disappears in the isothermal at Tc
d. Compressibility of vapour is infinite at Tc.
Ans. a
d. None of the above 60. Which one of the following statements related
Ans. * to Brownian motion is correct?
56. On a T-S diagram (y-axis : T; x-axis: S), what a. It is a result of combined effect of
are the slopes of adiabatics and isothermals chemical, electrical and gravitational
respectively ? forces.
a. Positive and negative b. Brownian speed increases if the vessel
b. Negative and positive containing gas is shaken violently.
c. Infinite and zero c. Brownian motion is faster if the viscosity
d. Zero and infinite is low.
Ans. c d. Brownian speed increases if the pressure
57. difference between end points of a
container is increased.
Ans. c
61. Starting from the enthalpy function (H) and
using the fact that dH is an exact differential,
we arrive at which Maxwells relation ?

Maxwell distribution functions (F) for ideal a.


gas molecules are plotted against their
velocities at two temperatures T1 and T2 as
shown above.
Which one of the following statements is not b.
correct ?
a. The most probable velocity for T1 is
greater than that for T2. c.
b. For T1, the spread of velocity distribution
is smaller.
c. Temperature T1 is lower than temperature d.
T2. Ans. b
d. Areas under the two curves should be 62. The distribution of radiant energy against
equal. wavelength is fully explained for whole
Ans. a spectral range by which one of the following
58. If the average speed of a gas molecule of mass laws ?
9kT a. Weins law
m at temperature T is , then what is the
m b. Rayleigh-Jeans law
average energy of the gas molecule ? c. Planck law
a. kT d. Maxwell distribution law
1 Ans. c
b. kT 63. If the temperature of a body increases, how
2
does the frequency corresponding to the
3
c. kT maximum radiant energy change ?
2 a. It increases exponentially
d. 2 kT b. It increases linearly
Ans. c c. It decreases linearly
59. Which one of the following statements is not, d. It decreases exponentially
correct in respect of the critical temperature Ans. b
(Tc) in Andrews experiment? 64.
8 of 14
68. A point charge Q is placed at one of
the corners of a cube. What is the total electric
flux passing through the cube?
a. Q/0
b. Q/(40)
c. Q/(60)
With reference to the figure given above, d. Q/(80)
which one of the following is correct ? Ans. d
a. T1 > T2 > T3 69. An isolated metallic sphere of radius 10 cm is
b. T1 < T2 < T3 given a charge of 10 cm. What is the force per
c. Dependence on temperature is significant unit area on the surface of the sphere?
only in the visible region of the spectrum 1
d. Dependence on temperature is significant = 9 x 109 Nm 2 C 2
4 0
only near the ultraviolet region of the
spectrum a. 3.6 N/m2
(The distributions follow Plancks law) b. 7.2 N/m2
Ans. b c. 3.6 x 10-8 N/m2
65. A hollow metallic sphere is given certain d. Zero
charge which is uniformly distributed over its Ans. d
surface. 70. The limit to the high potential which can, be
Consider the following statements: set by Van de Graaff generator is governed by
1. The field inside the cavity of the sphere is which of the following?
zero. 1. Insulation
2. The potential inside the cavity of then 2. Order of vacuum
sphere is zero. 3. Nature of the particle to be accelerated
3. The field at the surface of the sphere is Select the correct answer using the code given
normal to the surface. below:
Which of the statements given above is/are a. 1 only
correct? b. 1 and 2 only
a. 1 only c. 2 and 3 only
b. 3 only d. l, 2 and 3
c. 1, 2 and 3 Ans. b
d. 1 and 3 only 71. A parallel plate capacitor is charged to a
Ans. d potential V and then disconnected from the
66. The ratio of magnitude of electrostatic force battery. Now distance between the capacitor
and gravitational force between two protons plates is increased by pulling them apart.
separated by a distance is 1.2 x 1036 (e = 1.6 x Which physical quantity would not change ?
1 a. Potential difference between the plates of
10-19 C, mp = 1.67 x 10-27 kg, = 9 x 109 capacitor
40
b. Capacitance
Nm2C-2, G = 6.67 x 10-11 Nm2kg-2) c. Energy stored in the capacitor
If the distance between them is halved, what d. Electrical field between the plates of
will be the new ratio ? capacitor
a. 1.2 x 1036 Ans. d
b. 2.4 x 1036 72. Two parallel plates of a capacitor, separated by
c. 6.0 x 1035 2 mm, are at a potential difference of 20 V. A
d. 1.2 x 1030 dielectric sheet of dielectric constant 2 and
Ans. a thickness 1 mm is inserted between the plates.
67. A sphere with radius 8 m has a charge 3 C, How does the potential difference between the
uniformly distributed throughout the volume. two plates change?
What would be the electric field at a distance a. It increases by 5 V
40 m away from the centre of the sphere? b. It decreases by 5 V
1 c. It increases by 10 V
= 9 x 109 Nm 2 C2
4 0 d. It decreases by 10 V
Ans. b
a. 210 V/m
73. Two resistances R1 and R2, each of 100 , are
b. 120 V/m
connected in series with a battery of emf 2 V
c. 100 V/m
and negligible internal resistance. What is the
d. 70 V/m
value of the potential difference across the
Ans. a
9 of 14
resistance R1, as measured by a voltmeter c. A2, B1, C4, D3
having resistance of 1 k ? d. A4, B3, C2, D1
a. 1.8 V Ans. b
b. 0.1 V 78. Hall effect is an example of a direct
c. 0.9 V application of which one of the following
d. 2 V options ?
Ans. c
74.
a.

b.

c.
Consider the above figure
What is the effective resistance between the
terminals A and B ? d.
a. 5 Ans. d
b. 15 79.
c. 20
d. 25
Ans. a
75. What is the value of magnetic potential at any
point on the equatorial line of a bar magnet?
a. Zero
b. Infinite
c. Finite positive A uniformly charged parallel plate capacitor,
d. Finite negative with its plates parallel to x-z plane, is moving
Ans. a with a constant velocity towards x-axis, as
76. Temperature dependence of magnetic shown in the figure given above. What are the
susceptibility () of a material in the vicinity electric and magnetic fields in the space
between the two plates of the capacitor?
of transition temperature is shown below: G G G
a. E = jE y and B = 0
G G
b. E = jE y and B = kB
z
G G
c. E = jE y and B = kBz

G G G
d. E = 0 and B = kB
z

The above graph is indicative of what type Ans. b


a. Ferromagnetic 80.
b. Paramagnetic
c. Diamagnetic -I
d. Ferrimagnetic
Ans. d A copper disc is rotating steadily about its
77. Match List-I with List-II and select the correct centre in the uniform field between the two
answer using the code given below the lists poles of an electromagnet; the field is 900 to
List-I the plane of the disc as shown above. The
A. Soft magnetic material induced emf E between the centre O and any
B. Very hard magnetic material point on the rim R, varying with time t, can be
C. Non-ferromagnetic material represented by which graphical variation ?
D. Ferromagnetic material
List II
1. Large hysteresis loop
2. Small magnetic susceptibility a.
3. Large magnetic permeability
4. Small hysteresis loop
Code: b.
a. A2, B3, C4, D1
b. A4, B1, C2, D3
10 of 14
G
c. c. B = kcE
(
0 sin k x t
'
)
G
cos ( k x t )
E
d. B = k 0 '

d. c
Ans. a
85. In a cyclotron, which of the following
conditions are essential ?
Ans. d 1. A magnetic field bends the ions around, so
that they pass again and again through the
81. Which among the following can be the value same accelerating potential.
of current flowing though an inductance coil 2. The time required to complete one
when voltage across is the highest ? revolution in the magnetic field must be
a. 100 A d.c. the same whatever be the particle speed.
b. 10 A at 50 Hz 3. The frequency at which the ion circulates
c. 6 A at 100 Hz in the magnetic field must be greater than
d. 2 A at 500 Hz the frequency of the electric oscillator.
Ans. d Select the correct answer using the code given
82. In an AC circuit the input voltage and the below:
current are respectively given by the a. 1 and 2 only
expressions b. 2 and 3 only
V = 100 sin (100 t) V c. 1 and 3 only
d. 1, 2 and 3
I = 2 sin 100 t + A
2 Ans. a
What is the average power consumed in the 86. With an electromagnetic wave propagating in
circuit ? space, the total electromagnetic energy in a
a. 100 W certain volume V can be written as which
b. 200 W form?
c. 141 W a. The cross product of electric and magnetic
G G G
d. Zero vectors, i.e N = E x H
Ans. d b. The sum of electric energy density
83. 1 G G
U E = D.E and the magnetic energy
2
1 G G
density U B = B.H
2
c. Essentially the rate at which the electric
field is doing work on charges in the
GG
( )
volume V. i.e. E.J dV
A resistance R, an inductance L, a capacitance v
C, voltmeters V1, V2 and V3 are connected to d. Essentially the surface integral of
G
an alternating source of e.m.f. When the Poynting vector N over the surface which
frequency of the source is increased, what is G
the reading of voltmeter V3 at the resonant encloses the volume V, i.e. N.ds
frequency?
a. Equal to that of V1 Ans. c
b. Equal to that of V2 87. Which of the following four equations show
c. Equal to those of both V1 and V2 that (i) the magnetic field lines cannot start
d. Equal to readings of neither V1 nor V2 from a point nor end at a point, and (ii) the
Ans. a electrostatic field lines cannot form closed
84. The expression for the electric field of an loops?
electromagnetic wave is given by
G
E = jE 0 sin (k ' x t) 1.
What is the corresponding expression for the
magnetic induction of the wave ? 2.
G E
(
a. B = k 0 sin k ' x t
c
) 3.
G E
(
b. B = jk 0 sin k ' x t
c
) 4.
11 of 14
Select the correct answer using the code given 92. What type of a magnetic field is
below: needed in a Stern-Gerlach experiment to act
a. 2 and 1, respectively upon the beam of silver atoms ?
b. 2 and 3, respectively a. Highly uniform1
c. 4 and 3, respectively b. Non-uniform but slowly varying
d. 4 and 1, respectively c. Uniform but varying with time
Ans. b d. Highly non-uniform
88. An alkali metal plate has a work function of 40 Ans. b
eV. If incoming radiation of wavelength o 93. Taking into account the spins of the electron
leads to emission of electrons with energy 50 and the proton, for a total angular momentum
eV, what is the wavelength of the radiation (in quantum number F, into which one of the
terms of o) for which the electrons are following states will the ground state of
emitted with energy 20 eV ? hydrogen atom split ?
a. 2o/3 a. F = 1/2, 0
b. 2o b. F = 0, 1
c. 1.5 o c. F = 3/2, 1/2
d. 2.5 o d. F = 3/2, 1
Ans. c Ans. b
89. Upon which of the following parameters does 94. For the atomic states with principal quantum
the maximum energy of the photo electrons number N = 3, what will be the maximum
depend ? number of electrons in the N = 3, l =2 and 1=
1. The frequency of the radiation 1 states (l is the angular momentum quantum
2. The intensity of the radiation number)
3. The work function of the photo materials a. 2
Select the correct answer using the code given b. 8
below: c. 16
a. 1 and 2 only d. 20
b. 1 and 3 only Ans. c
c. 2 and 3 only 95. Many small droplets of mercury (Hg) form a
d. 1, 2 and 3 big drop. What happens to the final
Ans. b temperature of big drop ?
90. The longest wavelength of the radiation which a. It will decrease
leads to photo-ionization of hydrogen atom in b. It will increase
c. It will remain same
the ground state, is m. What is the wavelength
d. It will decrease first and then increase
of the radiation (in terms of m) emitted by
Ans. b
hydrogen atom when it undergoes transition
96. X-rays are reflected in first order from a
from N = 2 to N = 1 state, N being the
crystal at glancing angle of 300. Spacing
principal quantum number ?
between crystal planes is 0.3 nm. What will be
4
a. m the approximate wavelength of the diffracted
3 X-rays ?
3
b. m a. 3 3 / 10 nm
4
b. 3 3 / 20 nm
1
c. m c. 0.3 nm
4
d. 0.15 nm
2
d. m Ans. c
3 97. Match List-I with List-II and select the correct
Ans. a answer using the code given below the lists
91. What is the number of spectral lines emitted List-I
by a hydrogen atom excited to the nth energy (Experiment)
level ? A. Compton Effect
a. n B. Braggs law
b. n -1 C. Moseley plot
n ( n + 1) D. Stem-Gerlach experiment
c. List II
2
n ( n 1)
(Related Property)
d. 1. Space quantization
2 2. X-ray reflection
Ans. d
12 of 14
3. X-ray photo scattering 2. Field emission of electrons from
4. X-ray spectra a metal surface
Code: 3. -decay
a. A4, B1, C3, D2 Select the correct answer using the code given
b. A3, B2, C4, D1 below:
c. A4, B2, C3, D1 a. 1, 2 and 3
d. A3, B1, C4, D2 b. 1 only
Ans. b c. 1 and 2 only
98. Consider the following statements about d. 2 and 3 only
Comptons effect Ans. c
1. The wavelength of scattered X-rays is 103. Consider the following statements:
greater than that of incident X-rays. A particle of energy E is incident from the left
2. The wavelength of scattered X-rays is on a potential step of height V0. at x = 0;
smaller than that of incident X-rays. 1. if E < V0, wave function of the particle is
3. For a given value of scattering angle, the zero for x > 0.
shift in wavelength is independent of the 2. if E < V0, wave function is not zero for
wavelength of the incident radiation. 3. x > 0.
Which of the statements given above is/are 4. if E > V0, reflection coefficient is not
correct? zero.
a. 1 only Which of the statements given above is/are
b. 2 only correct?
c. 1 and 3 a. 1 only
d. 2 and 3 b. 2 only
Ans. c c. 1 and 3
99. Electrons with momentum p are incident at an d. 2 and 3
angle of 30 with the crystal surface. If d is the Ans. d
separation between crystal atomic planes 104. The 14C is an unstable nucleus. Which one of
parallel to the surface, what is the smallest the following correctly describes the -decay
value of p in terms of d and Plancks constant process for its decay ? (e is electron, e is
h, for which the reflected beam will show a positron, v is neutrino, and v is anti-neutrino.)
maximum intensity 7 a. 14C 14N + e + v
a. 2h/d b. 14C 14O + e + v
b. h/(2d)
c. 14C 14O + e + v
c. d/h
d. h/d d. 14C 14N + e + v
Ans. d Ans. d
100. A particle is described by a wave function 105. Which of the following particles are leptons ?
(x) - e-| x| in 1-dimension. What is the 1. Electron
probability that it will be found in the region 2. Pion
3. Neutron
|x| a, a > 0?
4. Neutrino
a. e-a
Select the correct answer using the code given
b. e-2a
below:
c. 1- e-a
a. 1 and 4 only
d. 1- e-2a
b. 1, 3 and 4
Ans. d
c. 2 and 3
101. In a collection of hydrogen atoms, the atoms
d. 1 and 2
are in different excited states. If they are
Ans. a
exposed to radiation with hv = 8 eV photonic
energies, what is the minimum energy of the 106. If m, m , mp are the masses of a -meson,
electrons coming out ? pi-meson and proton respectively, then which
a. 3.4 eV one of the following is correct ?
b. 4.6 eV a. m > mp > m
c. 6.6 eV b. m > m > mp
d. 8.0 eV c. mp > m > m
Ans. b d. mp > m > m
102. Which of the following phenomena can be Ans. c
explained by quantum mechanical tunnelling
through the barrier ?
1. -decay
13 of 14
107. b.

c.

If in a p-n junction diode a square input signal


of 8 V is applied (see figure above), what is
the value of the output signal across RL? d.

Ans. c
110. Which graph represents the nature of the curve
a.
between charge density p and distance r near
the depletion region of p-n junction diode?

b.

a.
c.

d. b.

Ans. c
c.
108. The diffusion constants Dn and Dp for
electrons and holes respectively are related to
their mobility by Einstein Equation as

d.
where VT is known as volt-equivalent of
temperature. What is VT equal to ?
kT Ans. c
a.
e 111.
k
b.
eT
eT
c.
k
e
d. What is the value of the diode current in the
kT circuit using germanium diode, shown above ?
(where the symbols have their usual meanings) a. 0 mA
Ans. a b. 0.5 mA
109. Which graph represents the voltage-current c. 1 mA
(V-I) characteristics of an ideal p-n junction d. 1.5 mA
diode ? Ans. a
112.

a.
14 of 14
In the transistor circuit given above, the base Reason (R): Because of diffraction,
current is 35 A. What is the value of the base the size of the image of a point is a disc of
resistor Rb ? roughly the size of the wavelength of the
a. 200 k imaging radiation.
b. 157 k Ans. a
c. 337 k 116. Assertion (A): At a very low pressure,
d. 257 k coefficient of viscosity increases with increase
Ans. d in density.
113. Inputs given to a logic gate are A and B and its Reason (R): At very low pressure, mean free
output is X. If A = 1, B = 0 then X 1. What path is comparable to the dimensions of the
type of gate this could be? container.
a. AND gate or NOR gate Ans. d
b. Or gate or NAND gate 117. Assertion (A): The change of internal energy
c. NOT gate oar NOR gate with respect to volume in free expansion of a
d. AND gate only perfect gas is zero
Ans. b
114. Ultraviolet light is used for erasing which
memory in a computer? Reason (R): This is an implication of the
a. RAM experimental observation of Joule that T= 0
b. EEPROM in such a process.
c. EPROM Ans. a
d. ROM 118. Assertion (A): Harmonic oscillator can not
Ans. c have zero energy.
Reason (R): Position and momentum donot
Directions : satisfy uncertainty relation.
Each of the next SIX items consists of two statements,
Ans. c
one labelled as the Assertion (A) and the other as 119. Assertion (A): When Zener diode is used in a
Reason (R). You are to examine these two statements d.c. voltage regulator, the output current and
carefully and select the answers to these items using hence the output voltage across the load
the codes given below: fluctuates.
Codes : Reason (R): The value of input voltage
(a) Both A and R are individually true and R is changes due to variation in a.c. supply voltage.
the correct explanation of A Ans. d
(b) Both A and R are individually true but R is 120. Assertion (A): Accumulator registers are
not the correct explanation of A general purpose registers.
(c) A is true but R is false Reason (R): This register (Accumulator) is
(d) A is false but R is true used as a destination of the results from ALU
operation.
115. Assertion (A) : Diffraction sets the limit to the
Ans. d
resolving power of all imaging systems.

Das könnte Ihnen auch gefallen